Hãy nhập câu hỏi của bạn vào đây, nếu là tài khoản VIP, bạn sẽ được ưu tiên trả lời.
câu a,mình ko biết nhưng câu b bạn cộng 1+b cho số hạng đầu áp dụng cô si,các số hạng khác tương tự rồi cộng vế theo vế,ta có điều phải c/m
\(1.\)\(a^3b^3\left(a^2-ab+b^2\right)\le\frac{\left(a+b\right)^8}{256}\)
\(\Leftrightarrow a^3b^3\left(a^2-ab+b^2\right)\left(a+b\right)\le\frac{\left(a+b\right)^9}{256}\)
\(\Leftrightarrow a^3b^3\left(a+b\right)^3\left(a^3+b^3\right)\le\frac{\left(a+b\right)^{12}}{256}\)
\(VT=ab\left(a+b\right).ab\left(a+b\right).ab\left(a+b\right).\left(a^3+b^3\right)\)
\(\le\left(\frac{ab\left(a+b\right)+ab\left(a+b\right)+ab\left(a+b\right)+\left(a^3+b^3\right)}{4}\right)^4\)
\(\le\frac{\left(a^3+3a^2b+3ab^2+b^3\right)^4}{256}\)
\(\le\frac{\left(a+b\right)^{12}}{256}\left(đpcm\right).\)
\(2.\) \(\frac{1}{1+a}+\frac{1}{1+b}+\frac{1}{1+c}\ge2\)
\(\Leftrightarrow\frac{1}{1+a}\ge1-\frac{1}{1+b}+1-\frac{1}{1+c}\)
\(\ge\frac{b}{1+b}+\frac{c}{1+c}\)
\(\ge2\sqrt{\frac{bc}{\left(1+b\right)\left(1+c\right)}}\)
\(\Rightarrow\hept{\begin{cases}\frac{1}{1+b}\ge2\sqrt{\frac{ac}{\left(1+a\right)\left(1+c\right)}}\\\frac{1}{1+c}\ge2\sqrt{\frac{ab}{\left(1+a\right)\left(1+b\right)}}\end{cases}}\)
\(\Rightarrow\frac{1}{1+a}.\frac{1}{1+b}.\frac{1}{1+c}\ge8\sqrt{\frac{a^2b^2c^2}{\left(1+a\right)^2.\left(1+b\right)^2.\left(1+c\right)^2}}\)\(\frac{1}{\left(1+a\right)\left(1+b\right)\left(1+c\right)}\ge\frac{8abc}{\left(1+a\right)\left(1+b\right)\left(1+c\right)}\)
\(\Leftrightarrow\) \(1\ge8abc\)
\(\Leftrightarrow\) \(abc\ge\frac{1}{8}\left(đpcm\right).\)
Ta có: \(a^2+2b+3=\left(a^2+1\right)+2\left(b+1\right)\ge2\left(a+b+1\right)\)
Tương tự ta có: \(b^2+2c+3\ge2\left(b+c+1\right)\); \(c^2+2a+3\ge2\left(c+a+1\right)\)
Từ đó suy ra\(\frac{a}{a^2+2b+3}+\frac{b}{b^2+2c+3}+\frac{c}{c^2+2a+3}\)\(\le\frac{a}{2\left(a+b+1\right)}+\frac{b}{2\left(b+c+1\right)}+\frac{c}{2\left(c+a+1\right)}\)
\(=\frac{1}{2}\left(\frac{a}{a+b+1}+\frac{b}{b+c+1}+\frac{c}{c+a+1}\right)\)
Đặt \(K=\frac{a}{a+b+1}+\frac{b}{b+c+1}+\frac{c}{c+a+1}\), ta đi chứng minh \(K\le1\)
Thật vậy: \(3-K=\frac{b+1}{a+b+1}+\frac{c+1}{b+c+1}+\frac{a+1}{c+a+1}\)
\(=\frac{\left(b+1\right)^2}{\left(b+1\right)\left(a+b+1\right)}+\frac{\left(c+1\right)^2}{\left(c+1\right)\left(b+c+1\right)}+\frac{\left(a+1\right)^2}{\left(a+1\right)\left(c+a+1\right)}\)
\(\ge\frac{\left(a+b+c+3\right)^2}{\left(b+1\right)\left(a+b+1\right)+\left(c+1\right)\left(b+c+1\right)+\left(a+1\right)\left(c+a+1\right)}\)(*)
Ta có: \(\left(b+1\right)\left(a+b+1\right)+\left(c+1\right)\left(b+c+1\right)+\left(a+1\right)\left(c+a+1\right)\)\(=3\left(a+b+c\right)+ab+bc+ca+a^2+b^2+c^2+3\)
(Mình gõ bằng chương trình Universal Math Solver, không hiện ảnh thì vô thống kê hỏi đáp của mình, ngày 30/5/2020 vào lúc 8:25)
\(=\frac{1}{2}\left[\left(a+b+c\right)^2+6\left(a+b+c\right)+9\right]=\frac{1}{2}\left(a+b+c+3\right)^2\)(**)
Từ (*) và (**) suy ra \(3-K\ge\frac{\left(a+b+c+3\right)^2}{\frac{1}{2}\left(a+b+c+3\right)^2}=2\Rightarrow K\le1\)
Vậy ta có điều phải chứng minh
Đẳng thức xảy ra khi a = b = c = 1
Áp dụng BĐT Cô-si,ta có :
\(a^2+1\ge2a\)
\(\Rightarrow\frac{a}{a^2+2b+3}\le\frac{a}{2a+2b+2}=\frac{1}{2}\left(\frac{a}{a+b+1}\right)\)
Tương tự : \(\frac{b}{b^2+2c+3}\le\frac{1}{2}\left(\frac{b}{b+c+1}\right);\frac{c}{c^2+2a+3}\le\frac{1}{2}\left(\frac{c}{c+a+1}\right)\)
\(\Rightarrow\frac{a}{a^2+2b+3}+\frac{b}{b^2+2c+3}+\frac{c}{c^2+2a+3}\le\frac{1}{2}\left(\frac{a}{a+b+1}+\frac{b}{b+c+1}+\frac{c}{c+a+1}\right)\)
Áp dụng BĐT Bu-nhi-a-cốp-ski,ta có :
\(\frac{a}{a+b+1}=\frac{a\left(a+b+c^2\right)}{\left(a+b+1\right)\left(a+b+c^2\right)}\le\frac{a^2+ab+ac^2}{\left(a^2+b^2+c^2\right)^2}=\frac{a^2+ab+ac^2}{9}\)
TT : ...
Cộng lại ta được :
\(\frac{a}{a+b+1}+\frac{b}{b+c+1}+\frac{c}{c+a+1}\le\frac{a^2+ab+ac^2}{9}+\frac{b^2+bc+ba^2}{9}+\frac{c^2+ca+cb^2}{9}\)
\(=\frac{a^2+b^2+c^2+ab+bc+ac+ac^2+ba^2+cb^2}{9}\le\frac{3+3+3}{9}=1\)
\(\Rightarrow\frac{a}{a^2+2b+3}+\frac{b}{b^2+2c+3}+\frac{c}{c^2+2a+3}\le\frac{1}{2}\)
Dấu "=" xảy ra khi a = b = c = 1
Áp dụng C-S
\(\frac{a}{2a+b+c}=\frac{a}{\left(a+b\right)+\left(a+c\right)}\le\frac{a}{4}\left(\frac{1}{a+b}+\frac{1}{a+c}\right)\)
b) chính là USAMO 2004. Đây là lời giải cung cấp bởi "http://www.artofproblemsolving.com/wiki/index.php/2004_USAMO_Problems/Problem_5"
Ta chứng minh được \(x^5+1\ge x^3+x^2\) suy ra \(x^5-x^2+3\ge x^3+2\).
Ta chỉ cần CM được \(\left(a^3+1+1\right)\left(1+b^3+1\right)\left(1+1+c^3\right)\ge\left(a+b+c\right)^3\)
Nhưng đây chính là BĐT Holder cho 3 bộ số mỗi bộ 3 số.
1. Vai trò a, b, c như nhau. Không mất tính tổng quát. Giả sử \(a\ge b\ge0\)
Mà \(ab+bc+ca=3\). Do đó \(ab\ge1\)
Ta cần chứng minh rằng \(\frac{1}{1+a^2}+\frac{1}{1+b^2}\ge\frac{2}{1+ab}\left(1\right)\)
Và \(\frac{2}{1+ab}+\frac{1}{1+c^2}\ge\frac{3}{2}\left(2\right)\)
Thật vậy: \(\left(1\right)\Leftrightarrow\frac{1}{1+a^2}-\frac{1}{1+ab}+\frac{1}{1+b^2}-\frac{1}{1+ab}\ge0\\ \Leftrightarrow\left(ab-a^2\right)\left(1+b^2\right)+\left(ab-b^2\right)\left(1+a^2\right)\ge0\\ \Leftrightarrow\left(a-b\right)\left[-a\left(1+b^2\right)+b\left(1+a^2\right)\right]\ge0\\ \Leftrightarrow\left(a-b\right)^2\left(ab-1\right)\ge0\left(BĐT:đúng\right)\)
\(\left(2\right)\Leftrightarrow c^2+3-ab\ge3abc^2\\ \Leftrightarrow c^2+ca+bc\ge3abc^2\Leftrightarrow a+b+c\ge3abc\)
BĐT đúng, vì \(\left(a+b+c\right)^2>3\left(ab+bc+ca\right)=q\)
và \(ab+bc+ca\ge3\sqrt[3]{\left(abc\right)^2}\)
Nên \(a+b+c\ge3\ge3abc\)
Từ (1) và (2) ta có \(\frac{1}{1+a^2}+\frac{1}{1+b^2}+\frac{1}{1+c^2}\ge\frac{3}{2}\)
Dấu ''='' xảy ra \(\Leftrightarrow a=b=c=1\)
Áp dụng BĐT Cauchy dạng \(\frac{9}{x+y+z}\le\frac{1}{x}+\frac{1}{y}+\frac{1}{z}\), ta được
\(\frac{9}{a+3b+2c}=\frac{1}{a+c+b+c+2b}\le\frac{1}{9}\left(\frac{1}{a+c}+\frac{1}{b+c}+\frac{1}{2b}\right)\)
Do đó ta được
\(\frac{ab}{a+3b+2c}\le\frac{ab}{9}\left(\frac{1}{a+c}+\frac{1}{b+c}+\frac{1}{2b}\right)=\frac{1}{9}\left(\frac{ab}{a+c}+\frac{ab}{b+c}+\frac{a}{2}\right)\)
Hoàn toàn tương tự ta được
\(\frac{bc}{2a+b+3c}\le\frac{1}{9}\left(\frac{bc}{a+b}+\frac{bc}{b+c}+\frac{b}{2}\right);\frac{ac}{3a+2b+c}\le\frac{1}{9}\left(\frac{ac}{a+b}+\frac{ac}{b+c}+\frac{c}{2}\right)\)
Cộng theo vế các BĐT trên ta được
\(\frac{ab}{a+3b+2c}+\frac{bc}{b+3c+2a}+\frac{ca}{c+3a+2b}\le\frac{1}{9}\left(\frac{ac+bc}{a+b}+\frac{ab+ac}{b+c}+\frac{bc+ab}{a+c}+\frac{a+b+c}{2}\right)=\frac{a+b+c}{6}\)Vậy BĐT đc CM
ĐẲng thức xảy ra khi và chỉ khi a = b = c >0
Lần sau đăng ít 1 thôi đăng nhiều ngại làm, bn đăng nhiều nên tui hướng dẫn sơ qua thôi tự làm đầy đủ vào vở
Bài 1:
Áp dụng BĐT AM-GM ta có:
\(a^4+b^4\ge2a^2b^2;b^4+c^4\ge2b^2c^2;c^4+a^4\ge2c^2a^2\)
Cộng theo vế 3 BĐT trên rồi thu gọn
\(a^4+b^4+c^4\ge a^2b^2+b^2c^2+c^2a^2\)
Áp dụng tiếp BĐT AM-GM
\(a^2b^2+b^2c^2=b^2\left(a^2+c^2\right)\ge2b^2ac\)
Tương tự rồi cộng theo vế có ĐPCM
Bài 2:
Quy đồng BĐT trên ta có:
\(\frac{a^2}{b^2}+\frac{b^2}{a^2}-\frac{a}{b}-\frac{b}{a}\ge0\)
\(\Leftrightarrow\frac{\left(a-b\right)^2\left(a^2+ab+b^2\right)}{a^2b^2}\ge0\) (luôn đúng)
Bài 4: Áp dụng BĐT AM-GM
\(a^3+b^3=\left(a+b\right)\left(a^2-ab+b^2\right)\)
\(\ge\left(a+b\right)\left(2ab-ab\right)=ab\left(a+b\right)\)
\(\Rightarrow\frac{a^3+b^3}{ab}\ge\frac{ab\left(a+b\right)}{ab}=a+b\)
Tương tự rồi cộng theo vế
Bài 5: sai đề tự nhien có dấu - :v nghĩ là +
ai k mình k lại [ chỉ 3 người đầu tiên mà trên 10 điểm hỏi đáp ]
B3 mk tìm đc cách giải r nhưng bạn nào muốn thì trả lời cg đc
Các bạn giải giúp mình B2 và B5 nhé. Mấy bài kia mình giải được rồi.
Bài 1. Từ giả thiết suy ra 1-a = b+c và áp dụng \(\left(x+y\right)^2\ge4xy\)
Ta có : \(4\left(1-a\right)\left(1-b\right)\left(1-c\right)=4\left(b+c\right)\left(1-c\right)\left(1-b\right)\le\left[\left(b+c\right)+\left(1-c\right)\right]^2\left(1-b\right)\)
\(=\left(b+1\right)^2\left(1-b\right)=\left(b+1\right)\left(1-b^2\right)=-b^2\left(b+1\right)+\left(b+1\right)\le b+1=a+2b+c\)